返回列表 发帖

GWD一道逻辑题求解释!!!

Which of the following, if true, provides evidence that most logically completes the argument below?
According to a widely held economic hypothesis, imposing strict environmental regulations reduces economic growth. This hypothesis is undermined by the fact that the states with the strictest environmental regulations also have the highest economic growth. This fact does not show that environmental regulations promote growth, however, since ______.
A. those states with the strictest environmental regulations invest the most in education and job training
B. even those states that have only moderately strict environmental regulations have higher growth than those with the least-strict regulations
C. many states that are experiencing reduced economic growth are considering weakening their environmental regulations
D. after introducing stricter environmental regulations, many states experienced increased economic growth
E. even those states with very weak environmental regulations have experienced at least some growth
这道题虽然我晓得是选A,可是不知道该怎么解释得通……
求哪位牛人给解答下!感激不尽啊O(∩_∩)O~
收藏 分享

其实这题是要你反驳“ environmental regulations promote growth

”这句话,所以看选项的时候发现:B环境保护更严厉的国家比不严厉的国

际经济增长快,明显是在支持不是在反驳。C,很多遭受了经济衰退的国家

正在考虑削弱环保规制,讲的是经济衰退的国家要干什么,没说削弱环保规

制后的效果是增长经济呢还是什么,无关项。D明显在支持,方向不对。E基

本没有环保规定的国家也有经济增长,跟要反驳的那句话完全不沾边。

TOP

恩啊~~~我也是排除选的A!可是想不通这个A是怎么削弱了呢……

TOP

This is a  weakening type question.

Common sense: Investment in Education and Job Training promotes growth in Economy.

A) simply proclaims that there is another reason to explain the economic growth. Whether investment in eduation and job training would lead to economic growth is not proven right or wrong. But it is a possible reason. That is enough to cast an unfavorable light on the hypothesis the author is trying to undermine.

There is another facet of the problem one needs to consider: the main conclusion.

The author agrees with the widely held hypothesis and he is trying to refute an undermining evidence (most strict regulation is associated with the highest growth.) His conclusion is that "This fact does not show that environmental regulations promote growth," and your job is to find a premise which supports this main conclusion.

TOP

Thx very much! Your explaination is so excellent!

TOP

这道题为什么不是C呢,我觉得c是因果倒置法,削弱了原题啊,题目中说environment regulation导致economic growth,而c中说因为reduced economic growth导致weak regulation

TOP

返回列表

站长推荐 关闭


美国top10 MBA VIP申请服务

自2003年开始提供 MBA 申请服务以来,保持着90% 以上的成功率,其中Top10 MBA服务成功率更是高达95%


查看